Free LSAT Prep Hour: Logical Reasoning; Necessary v. Sufficient Assumptions

Поділитися
Вставка
  • Опубліковано 24 жов 2024

КОМЕНТАРІ • 26

  • @itsbora
    @itsbora 3 роки тому +5

    He's such a great teacher. :) thank you!

  • @jamalari99
    @jamalari99 3 роки тому

    Not sure if this will help anyone, but Chelsea's question around 37:32, my answer is simply that the conclusion did not state the word "now," as in, Acme employees will "now" have a commute, it just states that they will. The word "now" would have made it a comparison to the present situation in Milltown, but it did not.

  • @DanielUmstead
    @DanielUmstead 4 роки тому +4

    Really appreciate the breakdown of D and E, I chose D and I was like okay good. And then Boom! E, premise does not support the conclusion and only focus on the current situation

  • @shevaschwartz
    @shevaschwartz 3 роки тому +2

    Thanks so much for this, it was very helpful!
    I have a question about the last example: isn’t the conclusion the fact that people weren’t surprised about the offence he took, rather than the fact that he took offence? So the sufficient assumption would have to relate to people’s perception of self-righteousness (as the correct answer did)? This was my understanding, so I wanted to check if I had misunderstood, even though I ended up at the same answer.

    • @manhattanpreplsat9493
      @manhattanpreplsat9493  3 роки тому

      That's 100% correct. The quick and dirty way to tackle a Sufficient Assumption question is to identify the new concept in the conclusion and eliminate any answer that doesn't address that concept. In this one, the new concept is "no one who knows him will be surprised" so the correct answer must deal with people's expectations or perceptions. Great job!

  • @roseb2105
    @roseb2105 Рік тому

    For question 1: would it also be reasonable to assume that the employees of acme will continue working for this company and not switch to one that is within a 30 minute or less compute from were they live?

  • @beaniejay6178
    @beaniejay6178 3 роки тому +1

    Thank you for the upload! I have a question for the Acme problem: If answer choice D said “more” instead of “less”, would that be a possible answer? I understand why we focused on “afford” since the question stem asked us to choose an answer that would strengthen the premise, but could “more” make us switch our perspective to the problem in terms of focusing on affordability -> to commute time before vs after? (If before the move the employees already had a 30 min commute, wouldn’t it be wrong to state that most have a 30-min commute only after the move?) Thank you!

    • @manhattanpreplsat9493
      @manhattanpreplsat9493  3 роки тому +1

      Hi there! To answer that question, first we have to take a look at how you paraphrased the conclusion of this argument. The argument actually concludes that once the company has moved, most employees will have a commute of more than 30 minutes. But in your question, you paraphrased that to "most will have a 30-minute commute only after the move." In doing so, you reversed the conditional logic of the statement. The actual conclusion is: Move --> Most > 30. Your paraphrase would be Most = 30 --> Move. Notice that there's also a subtle quantifier shift in your paraphrase. The actual conclusion said Most > 30. Yours said Most = 30. Now, if your paraphrase was the real conclusion, D, as written, would be a correct answer. In order to conclude that most will have a commute of 30 mins only after the move, it is necessary to assume that they currently have a commute of less than 30 minutes. When we see your suggestion to change "less" to "more," we think you were trying to correct that quantifier shift by saying it would be necessary assume that they don't currently have a commute of more than 30 minutes. In other words, they have a commute of 30 minutes of less. See the difference there? If you're going to reword it by changing "less" to "more," you also need to insert the "don't." So, big picture, if you were tempted by D, we're pretty sure you flip-flopped the conditional logic of the conclusion. And small picture, be careful with your quantifiers. If you want to represent "30 or less" using the word "more," you need to go with "no more than 30" or "they don't have more than 30." Hope this helps!

    • @beaniejay6178
      @beaniejay6178 3 роки тому

      @@manhattanpreplsat9493 Thank you so much for your detailed explanation! I really appreciate it 🥰🥰🥰

  • @sydneysears5432
    @sydneysears5432 3 роки тому +2

    Thank you Scott!! Super Helpful

  • @mrrockey96
    @mrrockey96 2 роки тому

    Thank you this was very helpful!

  • @roseb2105
    @roseb2105 Рік тому

    for question are you saying that answer choice A and D or incorrect beacuse a prediction about the future such as once a company moves to oceanview somthing will happen in this case employees commuting more that 3

  • @Asha21shell
    @Asha21shell 3 роки тому +1

    With the first assumption question where answer "E" was correct. Why could I not assume that even though the move was accompanied by a significant pay raise, the employees are still unable to afford housing with in 30 mins of the new location?

    • @manhattanpreplsat9493
      @manhattanpreplsat9493  3 роки тому +2

      Great question! We can't be certain that employees will be able to afford housing within 30 minutes if the move is accompanied by a pay raise. However, we don't have to be certain of that. To understand answer choice (E), we just have to recognize that a significant pay raise would cast doubt on the argument. The conclusion claims that most employees WILL have a commute of more than 30 minutes. It's a very strong claim. If the move is accompanied by a significant pay raise, employees might possibly be able to afford housing in Ocean View. We can no longer be certain that most of them WILL need to commute. In order for the original argument to hold up, it's necessary to assume that they won't receive a raise. This assumption eliminates a possible way for employees to move closer and avoid the commute. Hope that helps clarify!

    • @aminaguled2221
      @aminaguled2221 3 роки тому +2

      I also eliminated it for that reason. That question was the devil.

  • @conscious_competence3703
    @conscious_competence3703 3 роки тому +1

    Thanks for the vid.

  • @natasia64
    @natasia64 3 роки тому

    Defends argument against possible objection.

  • @Nat0528
    @Nat0528 3 роки тому

    I’m so happy I answered answer choice E on the first assumption question. Lol! I like the breakdown especially!

  • @mrquazimoto0075
    @mrquazimoto0075 Рік тому

    I like you.

  • @opinionatedreactions2102
    @opinionatedreactions2102 4 роки тому

    Why It say the “company bas moved”